User avatar
 
ManhattanPrepLSAT1
Thanks Received: 1909
Atticus Finch
Atticus Finch
 
Posts: 2851
Joined: October 07th, 2009
 
This post thanked 5 times.
 
 

Q17 - The stable functioning of a

by ManhattanPrepLSAT1 Sat Aug 14, 2010 3:58 pm

Great question! And the simple answer is no. A claim that is written explicitly can never be an assumption. An assumption is by definition unstated.

This question asks us to identify the role of the claim that a society is stable only if its laws tend to increase the happiness of its citizens.

The first sentence of this argument is the conclusion and can be identified by the words "this is clear from" that begin the second sentence. Everything after the first sentence is evidence for why the first sentence is true. Thus, answer choice (B) properly identifies the role of the claim.

(A) is not true, the first sentence is the conclusion.
(C) is not true, the claim does not challenge the conclusion, but rather supports it.
(D) could possibly be the correct answer to another argument, but the statement does not follow as a result of the conclusion, but rather supports the conclusion.
(E) would be accurate if the claim was an example of a general principle. In fact this claim is a general principle, not an illustration of one. We know this because of the words "only if."
 
clarafok
Thanks Received: 5
Forum Guests
 
Posts: 98
Joined: December 27th, 2010
 
 
trophy
Most Thankful
trophy
First Responder
 

Re: Q17 - The stable functioning of a

by clarafok Tue Jan 25, 2011 10:24 am

hmm i always get questions with 'general principle' as one of the answer choices wrong!

could you give examples of 'general principle' and 'an illustration of a general principle'?

thanks in advance!
User avatar
 
ManhattanPrepLSAT1
Thanks Received: 1909
Atticus Finch
Atticus Finch
 
Posts: 2851
Joined: October 07th, 2009
 
This post thanked 4 times.
 
 

Re: PT27, S1, Q17 - The stable functioning of

by ManhattanPrepLSAT1 Sun Jan 30, 2011 3:49 pm

Think of a principle as a general rule that can be applied to many specific situations.

Here's an example:

Anything difficult to understand is hard to learn. German is hard to understand, therefore German is hard to learn.



The first sentence represents a generality. The second and third sentences create an argument that relies on the general principle. The one tip I'd have for principle questions is to think of the principle in if ... then ... form. Then match the "if" part with the evidence and the "then" part with the conclusion. And of course, always keep in mind the contrapositive.
 
happyburst
Thanks Received: 0
Vinny Gambini
Vinny Gambini
 
Posts: 1
Joined: September 21st, 2014
 
 
 

Re: Q17 - The stable functioning of a

by happyburst Sun Sep 21, 2014 1:49 pm

A quick question on how to best approach the problem: as I'm reading the stimulus, I see a string of conditional statements. Would it be preferable to just read through the stimulus and question stem, or make a habit of diagramming the conditional statements as I read along (which takes up time and doesn't seem to be particularly necessary here)?
 
smsotolongo
Thanks Received: 1
Jackie Chiles
Jackie Chiles
 
Posts: 33
Joined: September 21st, 2014
 
 
 

Re: Q17 - The stable functioning of a

by smsotolongo Sun Jun 28, 2015 3:16 pm

mattsherman Wrote:(E) would be accurate if the claim was an example of a general principle. In fact this claim is a general principle, not an illustration of one. We know this because of the words "only if."


Could you elaborate a little more on this answer. I struggled with this answer and these types of questions where you have to think of the argument structurally and abstractly are killing me. Why can't we have more assumption family questions. :mrgreen: Thanks.
 
roflcoptersoisoi
Thanks Received: 0
Atticus Finch
Atticus Finch
 
Posts: 165
Joined: April 30th, 2015
 
 
 

Re: Q17 - The stable functioning of a

by roflcoptersoisoi Mon Jul 04, 2016 1:24 pm

We are asked to determine the function of the conditional statement in the third sentence, in the argument.
The stimulus is replete with conditionals so I took the liberty of drawing them out. You could also just try to use intuition to solve for this question.

1st sentence (Conclusion): Stable society (SC) then you need the long term goals of citizens to be stable (LTGCS)
2nd (Support): For a legislature to craft laws that augment the happines of citzens majority of citizens need to have long term goals that are stable.
3rd: Stable society (SC) --> laws augment happiness of citizens (AH)

3rd sentence (Premise): SC --> AH
2nd sentence (Premise) : LAH --> LTGCS)
------------------------------------------------------------
1st sentence: SC --> LTGCS (Conclusion)

We can see that when the conditional is combined to with the one in the second sentence, the conclusion can be properly drawn. Therefore we can see that the conditional in the third sentence is is a complimentary premise, that acts in tandem with the other premise to reach the conclusion.

In more abstract terms, this argument form takes the form of: A --> B B -->C, therefore A --> C, this is a valid argument form.

B--> C (premise, second sentence)
-------
A--> C (conclusion)

The third sentence gives us part of the missing logic chain (A --> B) and acts in tandem with the other sentence to substantiate the conclusion.


(A) No the first sentence is the conclusion
(B) Bingo. It's a complimentary premise which works in tandem with the other premise and when combined the conclusion is able to be properly drawn.
(C) No, it need not be refuted, when combined with the preceding sentence, it allows the conclusion to be properly drawn.
(D) It is not a consequence of the argument, it is part of the argument, it's a supporting premise.
(E) The argument isn't presupposing any general principle, the argument is actually valid, i.e., there is no defect in it's logic.
 
crocca
Thanks Received: 1
Vinny Gambini
Vinny Gambini
 
Posts: 18
Joined: August 01st, 2016
 
 
 

Re: Q17 - The stable functioning of a

by crocca Wed Jul 12, 2017 9:54 am

ManhattanPrepLSAT1 Wrote:Great question! And the simple answer is no. A claim that is written explicitly can never be an assumption. An assumption is by definition unstated."


Hello,

What was the question that you were answering here? The original post was deleted.

I understand why E is wrong on the grounds that the sentence is not an illustration of a general principle, but am still a little mixed up about the "presupposes". Can presuppositions not be explicitly stated?

Thank you in advance!